Problem maximized!

Algebra Level 4

x , y x,y and z z are positive reals satisfying x + y + z = 10 x+y+z = 10 .

If the maximum value of x y z + x y + y z + z x xyz + xy + yz + zx can be expressed as A B \dfrac AB , where A A and B B are coprime positive integers , find A + B A+B .


The answer is 1927.

This section requires Javascript.
You are seeing this because something didn't load right. We suggest you, (a) try refreshing the page, (b) enabling javascript if it is disabled on your browser and, finally, (c) loading the non-javascript version of this page . We're sorry about the hassle.

3 solutions

Rajen Kapur
Aug 29, 2016

Let us define new variables X = x + 1 , Y = y + 1 , Z = z + 1 X=x+1,Y=y+1,Z=z+1 to restate the problem as: Find the maximum value of ( X Y Z 11 ) (XYZ-11) when X + Y + Z = 13 X+Y+Z=13 . Using A.M.-G.M. inequality, it works out to ( 13 3 ) 3 11 = 1900 27 (\frac{13}{3})^3-11=\frac{1900}{27} valid for positive values of the variables. Hence answer 1927.

Did the same way! :-)

Akshat Sharda - 4 years, 9 months ago

I did the same way too.

Shreyash Rai - 4 years, 7 months ago
Chew-Seong Cheong
Aug 29, 2016

We can solve the problem by breaking the expression into two parts x y z xyz and x y + y z + z x xy+yz+zx and applying AM-GM inequality . Note that AM-GM inequality only applies if x , y , z > 0 x,y,z >0 , which is shown to be the case later.

x + y + z 3 x y z 3 x y z ( x + y + z 3 ) 3 = 1000 27 \begin{aligned} x+y+z & \ge 3\sqrt[3]{xyz} \\ \implies xyz & \le \left(\frac {x+y+z}3\right)^3 = \frac {1000}{27} \end{aligned}

Equality occurs when x = y = z = 10 3 > 0 x=y=z = \frac {10}3 > 0 .

x y + y z + z x ( x + y 2 ) 2 + ( y + z 2 ) 2 + ( z + x 2 ) 2 1 4 ( x 2 + 2 x y + y 2 + y 2 + 2 y z + z 2 + z 2 + 2 z x + x 2 ) 1 2 ( x 2 + y 2 + z 2 + x y + y z + z x ) 3 2 ( x y + y z + z x ) 1 2 ( x 2 + y 2 + z 2 + 2 ( x y + y z + z x ) ) 1 2 ( x + y + z ) 2 = 50 x y + y z + z x 2 3 50 = 100 3 \begin{aligned} xy+yz+zx & \le \left(\frac {x+y}2\right)^2 + \left(\frac {y+z}2\right)^2 + \left(\frac {z+x}2\right)^2 \\ & \le \frac 14 \left(x^2+2xy+y^2 + y^2+2yz+z^2 + z^2+2zx+x^2 \right) \\ & \le \frac 12 \left(x^2 + y^2 + z^2 + xy+yz+zx \right) \\ \implies \color{#D61F06}{\frac 32} (xy+yz+zx) & \le \frac 12 \left(x^2 + y^2 + z^2 + \color{#D61F06}{2}(xy+yz+zx) \right) \\ & \le \frac 12 (x+y+z)^2 = 50 \\ \implies xy+yz+zx & \le \frac 23 \cdot 50 = \frac {100}3 \end{aligned}

Equality again occurs when x = y = z = 10 3 x=y=z = \frac {10}3 . Therefore,

x y z + x y + y z + z x 1000 27 + 100 3 = 1900 27 \implies xyz + xy + yz + zx \le \dfrac {1000}{27} + \dfrac {100}3 = \dfrac {1900}{27}

A + Y = 1900 + 27 = 1927 \implies A+Y = 1900 + 27 = \boxed{1927}

I have a much simpler version for the maximum value of x y + y z + z x xy + yz + zx . By the Cauchy-Schwarz Inequality, we have, x 2 + y 2 + z 2 x y + y z + z x x^2+y^2+z^2 \geq xy + yz + zx . Adding 2 ( x y + y z + z x ) 2(xy + yz + zx ) to both sides, ( x + y + z ) 2 3 ( x y + y z + z x ) (x + y + z)^2 \geq 3(xy + yz + zx) . x y + y z + z x ( 10 ) 2 3 = 100 3 \implies xy +yz + zx \le \dfrac {(10)^2}{3} = \dfrac {100}{3} .

Harsh Poonia - 2 years, 10 months ago

Due to this problem is a calculus problem and the other solutions are algebraic solutions, I'll give a calculus solution

Relevant wiki: Lagrange multipliers

Let L ( x , y , z ) = x y z + x y + y z + z x + λ ( x + y + z 10 ) L(x,y,z) = xyz + xy + yz + zx + \lambda(x + y + z - 10) \Rightarrow d d x L ( x , y , z ) = y z + y + z + λ = 0 \dfrac{d}{dx} L(x,y,z) = yz + y + z + \lambda = 0 d d y L ( x , y , z ) = x z + x + z + λ = 0 \dfrac{d}{dy} L(x,y,z) = xz + x + z + \lambda = 0 d d z L ( x , y , z ) = y x + y + x + λ = 0. \dfrac{d}{dz} L(x,y,z) = yx + y + x + \lambda = 0. Substituing λ \lambda in the previous equations, we get y z + y + z = x z + x + z = y x + y + x yz + y + z = xz + x + z = yx + y + x x = y = z \Rightarrow x = y = z and substituing in x + y + z = 10 x + y + z = 10 we get x = y = z = 10 3 x = y = z = \frac{10}{3} .

We can see that the hessian matrix d 2 L ( 10 / 3 , 10 / 3 , 10 / 3 ) d^2 L(10/3,10/3,10/3) is a defined negative matrix(it's left to the reader as an exercise), so x y z + x y + y z + z x xyz + xy +yz + zx will have a maximum for x = y = z = 10 3 x = y = z = \frac{10}{3} and this maximum is ( 10 3 ) 3 + 3 ( 10 3 ) 2 = 1000 27 + 100 3 = 1900 27 (\frac{10}{3})^3 + 3(\frac{10}{3})^2 = \frac{1000}{27} + \frac{100}{3} = \frac{1900}{27} ...

0 pending reports

×

Problem Loading...

Note Loading...

Set Loading...